Đến nội dung

Hình ảnh

Cho $a,b,c\geq 0$ a+b+c=1. Tìm max, min: $\frac{1}{a^{2}+1}+\frac{1}{b^{2}+1}+\frac{1}{c^{2}+1}$


  • Please log in to reply
Chủ đề này có 3 trả lời

#1
ngoc980

ngoc980

    Hạ sĩ

  • Thành viên
  • 84 Bài viết
Cho $a,b,c\geq 0$ a+b+c=1. Tìm max, min: $\frac{1}{a^{2}+1}+\frac{1}{b^{2}+1}+\frac{1}{c^{2}+1}$

Đừng để những khó khăn đánh gục bạn, hãy kiên nhẫn rồi bạn sẽ vượt qua.

Đừng chờ đợi những gì bạn muốn mà hãy đi tìm kiếm chúng.


#2
BlackSelena

BlackSelena

    $\mathbb{Sayonara}$

  • Hiệp sỹ
  • 1549 Bài viết

Cho $a,b,c\geq 0$ a+b+c=1. Tìm max, min: $\frac{1}{a^{2}+1}+\frac{1}{b^{2}+1}+\frac{1}{c^{2}+1}$

Min trước hè :P.
$$\frac{1}{a^{2}+1}+\frac{1}{b^{2}+1}+\frac{1}{c^{2}+1}$$
Ta có: $$\frac{1}{b^2+1} = a - \frac{ab^2}{b^2+1} \geq a - \frac{ab}{2}$$
Thiết lập các bđt tương tự, ta có:
$$VT \geq a+b+c - \frac{ab+bc+ca}{2}$$
Mặt khác ta có bđt cơ bản sau $(a+b+c)^2 \geq 3(ab+bc+ca)$
$\Rightarrow ab+bc+ca \leq \frac{1}{3}$
Vậy $$a+b+c - \frac{ab+bc+ca}{2} \geq 1 - \frac{1}{6} = \frac{5}{6}$$
Đẳng thức xảy ra khi $a=b=c = \frac{1}{3}$

Bài viết đã được chỉnh sửa nội dung bởi BlackSelena: 25-09-2012 - 21:05


#3
ngoc980

ngoc980

    Hạ sĩ

  • Thành viên
  • 84 Bài viết

Min trước hè :P.
$$\frac{1}{a^{2}+1}+\frac{1}{b^{2}+1}+\frac{1}{c^{2}+1}$$
Ta có: $$\frac{1}{b^2+1} = a - \frac{ab^2}{b^2+1} \geq a - \frac{ab}{2}$$

Hình như chỗ này sai rồi ấy

$a-\frac{1}{b^{2}+1}=\frac{ab^{2}+a-1}{b^{2}+1}\neq \frac{ab^{2}}{b^{2}+1}$bạn xem lại xem :mellow: :wacko:
_______
Híc híc nhầm đề mất tiêu :P, sò zý

Bài viết đã được chỉnh sửa nội dung bởi BlackSelena: 25-09-2012 - 22:08


Đừng để những khó khăn đánh gục bạn, hãy kiên nhẫn rồi bạn sẽ vượt qua.

Đừng chờ đợi những gì bạn muốn mà hãy đi tìm kiếm chúng.


#4
dark templar

dark templar

    Kael-Invoker

  • Hiệp sỹ
  • 3788 Bài viết

Cho $a,b,c\geq 0$ a+b+c=1. Tìm max, min: $\frac{1}{a^{2}+1}+\frac{1}{b^{2}+1}+\frac{1}{c^{2}+1}$

Bài này xài mẹo vặt tý :D
Viết lại biểu thức $A=\frac{1}{a^2+1}+\frac{1}{b^2+1}+\frac{1}{c^2+1}$ :
$$A=3-\left(\frac{a^2}{a^2+1}+\frac{b^2}{b^2+1}+\frac{c^2}{c^2+1} \right)$$
Theo AM-GM:
$$\frac{a^2}{a^2+1} \le \frac{a}{2}$$
Nên :
$$A \ge 3-\frac{a+b+c}{2}=\frac{5}{2}$$
Vậy $A_{\min}=\frac{5}{2} \iff (a;b;c) \sim (0;0;1)$.
Giờ tìm max :
Ta sẽ chứng minh rằng:
$$\frac{1}{a^2+1}+\frac{1}{b^2+1}+\frac{1}{c^2+1} \le \frac{27}{10}(*)$$
Theo cách đặt $p,q,r$;ta có:
$$(*) \iff 27r^2-34r+17q^2-14q+4 \ge 0$$
Dễ dàng có :$34r \le \frac{34}{27}$ nên ta chỉ cần chứng minh:
$$27r^2+17q^2-14q+\frac{74}{27} \ge 0$$
Theo Schur:
$$27r^2 \ge \frac{(4q-1)^2}{3}$$
Nên:
$$27r^2+17q^2-14q+\frac{74}{27} \ge \frac{(4q-1)^2}{3}+17q^2-14q+\frac{74}{27}=(3q-1)(201q-83) \ge 0;\forall q \le \frac{1}{3}$$
Vậy $A_{\max}=\frac{27}{10} \iff a=b=c=\frac{1}{3}$.
"Do you still... believe in me ?" Sarah Kerrigan asked Jim Raynor - Starcraft II:Heart Of The Swarm.




1 người đang xem chủ đề

0 thành viên, 1 khách, 0 thành viên ẩn danh